Last visit was: 24 Apr 2024, 22:55 It is currently 24 Apr 2024, 22:55

Close
GMAT Club Daily Prep
Thank you for using the timer - this advanced tool can estimate your performance and suggest more practice questions. We have subscribed you to Daily Prep Questions via email.

Customized
for You

we will pick new questions that match your level based on your Timer History

Track
Your Progress

every week, we’ll send you an estimated GMAT score based on your performance

Practice
Pays

we will pick new questions that match your level based on your Timer History
Not interested in getting valuable practice questions and articles delivered to your email? No problem, unsubscribe here.
Close
Request Expert Reply
Confirm Cancel
SORT BY:
Date
Tags:
   
Show Tags
Hide Tags
Tutor
Joined: 16 Oct 2010
Posts: 14817
Own Kudos [?]: 64906 [1]
Given Kudos: 426
Location: Pune, India
Send PM
Manager
Manager
Joined: 17 Jul 2017
Posts: 205
Own Kudos [?]: 93 [0]
Given Kudos: 228
Send PM
Current Student
Joined: 04 Jun 2018
Posts: 142
Own Kudos [?]: 66 [1]
Given Kudos: 139
GMAT 1: 710 Q50 V36
GMAT 2: 690 Q50 V32
GMAT 3: 610 Q48 V25
Send PM
Current Student
Joined: 04 Jun 2018
Posts: 142
Own Kudos [?]: 66 [0]
Given Kudos: 139
GMAT 1: 710 Q50 V36
GMAT 2: 690 Q50 V32
GMAT 3: 610 Q48 V25
Send PM
Re: Veritas Prep CR Forum Expert - Karishma - Ask Me Anything about CR [#permalink]
The retinas of premature babies are not fully developed at birth. Because their retinas are so delicate, premature babies sometimes lose their sight. Methods for preventing this syndrome, which is called retinopathy of prematurity, have improved, but the proportion of premature babies who lose their sight because of this syndrome has increased.

Which of the following, if true, best reconciles the apparent discrepancy described above?

A. When premature babies are born, their retinas are developmentally unprepared to deal with light and air in the environment outside the womb.
B. The oxygen that must be administered to premature babies at birth can sometimes have a damaging effect on the babies’ retinas, but the oxygen is now administered in less damaging concentrations than it used to be.
C. The effects of retinopathy of prematurity can be reduced by controlling the exposure of premature babies to light and oxygen, but this method cannot completely prevent the syndrome.
D. The improvement of methods to prevent retinopathy of prematurity has been a gradual process, and there is still a need for further knowledge.
E. Improved medical technology is saving the lives of premature babies who would previously have died, but these babies have even more delicate retinas than do other premature babies and are more apt to lose their sight.


HI VeritasKarishma
The OA for this question is B.
But I cannot understand how does it resolve the paradox.
Paradox according to me:
THe methods have improved----which should have led to the fall in the incidence of premature babies losing their sight

But the proportion of premature babies who lost sight has increased.

I pre thought:
What if the number of premature babies has reduced. IN this case both of the facts can be true.


But the answer option B only addressed the aspect that the methods have improved and not how the proportion has increased.


Looking forward to your reply!

Regards
Nitesh
Tutor
Joined: 16 Oct 2010
Posts: 14817
Own Kudos [?]: 64906 [4]
Given Kudos: 426
Location: Pune, India
Send PM
Re: Veritas Prep CR Forum Expert - Karishma - Ask Me Anything about CR [#permalink]
2
Kudos
2
Bookmarks
Expert Reply
The answer is not (B) here. It is (E). What is the source of this question?

Paradox: Retinopathy methods have improved but the proportion of premature babies who lose their sight has increased.

This is certainly a paradox. If methods have improved, the proportion of babies losing their sight should have decreased.

How do we reconcile it?

Look at option (E)

E. Improved medical technology is saving the lives of premature babies who would previously have died, but these babies have even more delicate retinas than do other premature babies and are more apt to lose their sight.

Better technology is saving lives of babies who would have died (say extremely premature born in 5th-6th month). Their retinas are even more delicate so even with better methods, it is difficult to save their sight. These babies would have died earlier so they would not have been counted in the "proportion who lose their sight". Now, since they are saved, but their retinas are extremely delicate, they end up getting counted in the total but their sight is lost. So overall, the proportion of those who lose their sight is increasing. This explains why despite better retinotherapy, proportion of babies losing sight is increasing. Now it all makes sense.

(B) does not explain the paradox.
B. The oxygen that must be administered to premature babies at birth can sometimes have a damaging effect on the babies’ retinas, but the oxygen is now administered in less damaging concentrations than it used to be.

If less damaging doses of oxygen are administered now, proportion of babies whose sight is saved should have increased. This doesn't help us resolve why the proportion has decreased.



nitesh50 wrote:
The retinas of premature babies are not fully developed at birth. Because their retinas are so delicate, premature babies sometimes lose their sight. Methods for preventing this syndrome, which is called retinopathy of prematurity, have improved, but the proportion of premature babies who lose their sight because of this syndrome has increased.

Which of the following, if true, best reconciles the apparent discrepancy described above?

A. When premature babies are born, their retinas are developmentally unprepared to deal with light and air in the environment outside the womb.
B. The oxygen that must be administered to premature babies at birth can sometimes have a damaging effect on the babies’ retinas, but the oxygen is now administered in less damaging concentrations than it used to be.
C. The effects of retinopathy of prematurity can be reduced by controlling the exposure of premature babies to light and oxygen, but this method cannot completely prevent the syndrome.
D. The improvement of methods to prevent retinopathy of prematurity has been a gradual process, and there is still a need for further knowledge.
E. Improved medical technology is saving the lives of premature babies who would previously have died, but these babies have even more delicate retinas than do other premature babies and are more apt to lose their sight.


HI VeritasKarishma
The OA for this question is B.
But I cannot understand how does it resolve the paradox.
Paradox according to me:
THe methods have improved----which should have led to the fall in the incidence of premature babies losing their sight

But the proportion of premature babies who lost sight has increased.

I pre thought:
What if the number of premature babies has reduced. IN this case both of the facts can be true.


But the answer option B only addressed the aspect that the methods have improved and not how the proportion has increased.


Looking forward to your reply!

Regards
Nitesh
Director
Director
Joined: 16 Sep 2016
Status:It always seems impossible until it's done.
Posts: 645
Own Kudos [?]: 2055 [0]
Given Kudos: 174
GMAT 1: 740 Q50 V40
GMAT 2: 770 Q51 V42
Send PM
Re: Veritas Prep CR Forum Expert - Karishma - Ask Me Anything about CR [#permalink]
Dear VeritasKarishma,

Can you please take a look at this question. - https://gmatclub.com/forum/the-water-te ... 03369.html. I could arrive at the OA by elimination and also the logic that since the temperature of water in the ocean does not fluctuate much and "Most animals can tolerate only a limited range of all the various conditions that exist in an estuary" hence the oceans are more favorable at least in terms of constant temp. However, this seems like a very weak reasoning for an inference type question.

Please help.

Thanks as always. :-)

Best,
Gladi

Q for reference:
Quote:
The water temperature in an estuary is influenced by fluctuations in air temperature to a much greater extent than is the case in the ocean. If the estuary is shallow, and the fresh and saline components are well mixed, temperature and salinity are usually the same from top to bottom, although temperatures in the shallow waters over mud flats may rise considerably in the summer. Oxygen and carbon dioxide levels in the water vary with photosynthesis, respiration, and decomposition of organic materials. Most animals can tolerate only a limited range of all the various conditions that exist in an estuary; thus each species selects that portion of an estuary where the normal changes in temperature, salinity, and the like fall within its range.


The most logical conclusion to draw from this paragraph is that

oxygen and carbon dioxide levels in an estuary change significantly depending on the latitude of the wetlands
the life forms found in the ocean are probably greater in number as compared to those found in an estuary
water temperature in an estuary compared to a stagnate water mass—a pond, for example— would have a profound impact on the ecological surroundings
photosynthesis, respiration, and decomposition of organic materials are common to all waterrelated habitats
evidence demonstrates that species of animals found in an estuary probably evolved from the ocean
Tutor
Joined: 16 Oct 2010
Posts: 14817
Own Kudos [?]: 64906 [1]
Given Kudos: 426
Location: Pune, India
Send PM
Re: Veritas Prep CR Forum Expert - Karishma - Ask Me Anything about CR [#permalink]
1
Kudos
Expert Reply
nitesh50 wrote:

Hi Karishma.


SO I have been struggling with causation for a long time now. I often get confused when we have decide whether an alternate cause can actually become a weakner or not.

1.Journalist: In physics journals, the number of articles reporting the results of experiments involving particle accelerators was lower last year than it had been in previous years. Several of the particle accelerators at major research institutions were out of service the year before last for repairs, so it is likely that the low number of articles was due to the decline in availability of particle accelerators.

Which of the following, if true, most seriously undermines the journalist’s argument?

Recent changes in the editorial policies of several physics journals have decreased the likelihood that articles concerning particle-accelerator research will be accepted for publication.

2. A study of marital relationships in which one partner's sleeping and waking cycles differ from those of other partner reveals that such couples share fewer activities with each other and have more violent arguments than do couples in a relationship in which both partners follow the same sleeping and waking patterns . Thus, mismatched sleeping and waking cycles can seriously jeopardize a marriage.

Which of the following, if true, most seriously weakens the argument above?

Married couples in which both spouses follow the same sleeping and waking patterns also have arguments that can jeopardize the couple's marriage.

3. 12 years ago and again 5 years ago, there were extended periods when the Darfir Republic's currency, the pundra, was weak: its value was unusually low relative to the world's most stable currencies. Both times a weak pundra made Darfir's manufactured products a bargain on world markets, and Darfir's exports were up substantially. Now some politicians are saying that, in order to cause another similarly sized increase in exports, the government should allow the pundra to become weak again.

Which of the following, if true, provides the government with the strongest grounds to doubt that the politicians' recommendation, if followed, will achieve its aim?

A sharp improvement in the efficiency of Darfir's manufacturing plants would make Darfir's product a bargain on world markets even without any weakening of the pundra relative to other currencies.




In case 1 the causation implies a past event.
Hence the alternate cause is actually a weakner.



Thus, mismatched sleeping and waking cycles can seriously jeopardize a marriage.

In case 2 the causation is more like a characteristic,
Hence the alternate cause is not a weakner.

Also, we can say that the argument says that
A and B occur together and then assumes that A causes B.
Hence the alternate cause will not be a weakner.



The government should allow the pundra to become weak again.

In case 3 the causation relates to a future event
Hence the alternate cause is not a weakner.




Now in the mood elevation question:

Conclusion:
sugars can play a major role in mood elevation.

Can we say that this option is more like case 2. Since it says "can", the alternate cause is not a weakner.
But, if we were to drop can and say that
Sugars play a major role in mood elevation,
in this case Only A causes B will be an assumption.


Looking forward to your analysis.


Regards
Nitesh


I don't understand your logic of past vs future. It doesn't have anything to do with it. Consider each case to be a discussion between two people.

Case 1:

A: Number of articles reporting the results of experiments involving particle accelerators was lower last year than it had been in previous years. It is because many were out of service.
B: Not necessary. Recent changes in the editorial policies of several physics journals have decreased the likelihood that articles concerning particle-accelerator research will be accepted for publication.
B has given an alternative reason for lower number of articles. Perhaps, the accelerators being out of service didn't have much to do with it. Perhaps they were still available when needed. But since magazines are printing fewer such articles, we saw fewer. It makes sense. It weakens the claim of A that the reason was X. There was another factor Y which could have been responsible.

Case 2:
A: Couples with different sleeping patterns share fewer activities with each other and have more violent arguments than do couples in which both partners follow the same sleeping and waking patterns. Thus, mismatched sleeping and waking cycles can seriously jeopardize a marriage.
B: Couples with same patterns also have arguments that can jeopardise marriage.

A already says that couples with different sleeping patterns have MORE violent arguments than do those in which both partners have same routine. A doesn't say that couples with same sleeping patterns don't have arguments. Hence, it doesn't weaken A's argument.

Case 3:
A's plan: In order to cause another similarly sized increase in exports, the government should allow the pundra to become weak again

We need to find a reason the plan won't work. So a reason why even if we let tundra become weak, we will not see similarly sized increase in exports.
Now here, does it help to say that a similar increase can be obtained in another way? No. We need to find a reason why our plan won't work. The fact that another plan will work is not relevant.

In the mood elevation question, we are looking for an assumption. We are not looking for a weakener. An assumption is what connects the premises to the conclusion. Hence I am not sure how you are applying this concept to the mood elevation question.
Tutor
Joined: 16 Oct 2010
Posts: 14817
Own Kudos [?]: 64906 [2]
Given Kudos: 426
Location: Pune, India
Send PM
Re: Veritas Prep CR Forum Expert - Karishma - Ask Me Anything about CR [#permalink]
2
Kudos
Expert Reply
Gladiator59 wrote:
Dear VeritasKarishma,

Can you please take a look at this question. - https://gmatclub.com/forum/the-water-te ... 03369.html. I could arrive at the OA by elimination and also the logic that since the temperature of water in the ocean does not fluctuate much and "Most animals can tolerate only a limited range of all the various conditions that exist in an estuary" hence the oceans are more favorable at least in terms of constant temp. However, this seems like a very weak reasoning for an inference type question.

Please help.

Thanks as always. :-)

Best,
Gladi


Here you go Gladi: https://gmatclub.com/forum/the-water-te ... l#p2192112
Director
Director
Joined: 16 Sep 2016
Status:It always seems impossible until it's done.
Posts: 645
Own Kudos [?]: 2055 [0]
Given Kudos: 174
GMAT 1: 740 Q50 V40
GMAT 2: 770 Q51 V42
Send PM
Re: Veritas Prep CR Forum Expert - Karishma - Ask Me Anything about CR [#permalink]
Dear VeritasKarishma,

Can you please look at this one - https://gmatclub.com/forum/u-s-environm ... 01438.html

I was not able to choose between B and E. Also some expert opinions seem to be divided. Your take on it would be valuable to the community.

Thanks as always.

Regards,
Gladi

Posted from my mobile device
Tutor
Joined: 16 Oct 2010
Posts: 14817
Own Kudos [?]: 64906 [1]
Given Kudos: 426
Location: Pune, India
Send PM
Re: Veritas Prep CR Forum Expert - Karishma - Ask Me Anything about CR [#permalink]
1
Kudos
Expert Reply
Gladiator59 wrote:
Dear VeritasKarishma,

Can you please look at this one - https://gmatclub.com/forum/u-s-environm ... 01438.html

I was not able to choose between B and E. Also some expert opinions seem to be divided. Your take on it would be valuable to the community.

Thanks as always.

Regards,
Gladi

Posted from my mobile device


Gladi,

See if this helps: https://gmatclub.com/forum/u-s-environm ... l#p2193472
Director
Director
Joined: 21 Jun 2017
Posts: 638
Own Kudos [?]: 530 [0]
Given Kudos: 4092
Location: India
Concentration: Finance, Economics
GMAT 1: 660 Q49 V31
GMAT 2: 620 Q47 V30
GMAT 3: 650 Q48 V31
GPA: 3.1
WE:Corporate Finance (Non-Profit and Government)
Send PM
Re: Veritas Prep CR Forum Expert - Karishma - Ask Me Anything about CR [#permalink]
https://gmatclub.com/forum/the-people-m ... 11268.html

Hi VeritasKarishma

Got torn between B and D.
Rejected B : People not watching TV debates are out of the scope of passage.
Chose D : Unpredictable behaviour will effect the likelihood of a win (Improve or Decrease) and thus, this is the correct weakener.


Another issue i am facing is that I am not able to recognize the patterns in a CR question. For example this was a causal pattern and in haste or maybe in anxiety I can never figure out if it is a causal pattern or a sampling pattern etc. Any pointers ?



Regards
Director
Director
Joined: 16 Sep 2016
Status:It always seems impossible until it's done.
Posts: 645
Own Kudos [?]: 2055 [0]
Given Kudos: 174
GMAT 1: 740 Q50 V40
GMAT 2: 770 Q51 V42
Send PM
Re: Veritas Prep CR Forum Expert - Karishma - Ask Me Anything about CR [#permalink]
Dear VeritasKarishma,

Whenever I run into CR trouble I turn to you. :-)

Can you please comment on this one - https://gmatclub.com/forum/in-a-recent- ... l#p2194617 . I am in a dilemma between option C vs. D

As always, you're the best. Thank you.

Regards,
Gladi
Manager
Manager
Joined: 17 Jul 2017
Posts: 205
Own Kudos [?]: 93 [0]
Given Kudos: 228
Send PM
Re: Veritas Prep CR Forum Expert - Karishma - Ask Me Anything about CR [#permalink]
Mam ,please evaluate my ESR.I donot know how to improve .
https://gmatclub.com/forum/request-to-e ... l#p2194651
Tutor
Joined: 16 Oct 2010
Posts: 14817
Own Kudos [?]: 64906 [0]
Given Kudos: 426
Location: Pune, India
Send PM
Re: Veritas Prep CR Forum Expert - Karishma - Ask Me Anything about CR [#permalink]
Expert Reply
ShankSouljaBoi wrote:
https://gmatclub.com/forum/the-people-most-likely-to-watch-a-televised-debate-between-political-111268.html

Hi VeritasKarishma

Got torn between B and D.
Rejected B : People not watching TV debates are out of the scope of passage.
Chose D : Unpredictable behaviour will effect the likelihood of a win (Improve or Decrease) and thus, this is the correct weakener.


Another issue i am facing is that I am not able to recognize the patterns in a CR question. For example this was a causal pattern and in haste or maybe in anxiety I can never figure out if it is a causal pattern or a sampling pattern etc. Any pointers ?


Regards


Note what option (D) says: Comments affect the behaviour (not "winning" affects the behaviour). Also, whether they increase likelihood of winning, we don't know.
Hence (D) is not correct.

https://gmatclub.com/forum/the-people-m ... l#p2194802
Tutor
Joined: 16 Oct 2010
Posts: 14817
Own Kudos [?]: 64906 [3]
Given Kudos: 426
Location: Pune, India
Send PM
Re: Veritas Prep CR Forum Expert - Karishma - Ask Me Anything about CR [#permalink]
2
Kudos
1
Bookmarks
Expert Reply
ShankSouljaBoi wrote:
https://gmatclub.com/forum/the-people-most-likely-to-watch-a-televised-debate-between-political-111268.html

Hi VeritasKarishma

Got torn between B and D.
Rejected B : People not watching TV debates are out of the scope of passage.
Chose D : Unpredictable behaviour will effect the likelihood of a win (Improve or Decrease) and thus, this is the correct weakener.


Another issue i am facing is that I am not able to recognize the patterns in a CR question. For example this was a causal pattern and in haste or maybe in anxiety I can never figure out if it is a causal pattern or a sampling pattern etc. Any pointers ?



Regards


ShankSouljaBoi,

As for recognising patterns - don't try to fit everything in one template or another. All I ever do is identify the type of question (weaken here) and then focus on conclusion and try to weaken that. The more complicated you make it, higher the chances of error. Just use logic. I don't believe in methodologies that make you dissect the reasoning followed in every argument. It wastes a lot of time, is unnecessary (until and unless it is a "method of reasoning" question) and doesn't necessarily lead to correct answer. What is to be gained?
Manager
Manager
Joined: 01 May 2018
Posts: 56
Own Kudos [?]: 103 [0]
Given Kudos: 80
Send PM
Re: Veritas Prep CR Forum Expert - Karishma - Ask Me Anything about CR [#permalink]
Hello VeritasKarishma
Thank you for all these initiatives for helping us out. I wanted your assistance in evaluate questions. My current approach to Evaluate question is that the correct choice when true would strengthen and when not true would weaken the argument. But I have encountered questions in which the correct choice just strengthens the argument but the negation doesnt weaken.
https://gmatclub.com/forum/truesave-is- ... 72955.html is an example. In this question, my instincts said C must be the answer but following the process, I wasnt able to reach the answer. In addition, A also seemed a strong contender. Request your help on this.
Senior Manager
Senior Manager
Joined: 20 Mar 2018
Posts: 476
Own Kudos [?]: 352 [0]
Given Kudos: 149
Location: Ghana
Concentration: Finance, Statistics
GMAT 1: 710 Q49 V39
Send PM
Re: Veritas Prep CR Forum Expert - Karishma - Ask Me Anything about CR [#permalink]
Hello Karishma,
Please is it a good idea to start reading the question stem before reading the stimuli or argument or vice versa or is there any strategy which will make you answer very fast and accurate?

Thank you


Sent from my iPhone using GMAT Club Forum mobile app
Manager
Manager
Joined: 12 Jul 2017
Posts: 199
Own Kudos [?]: 212 [0]
Given Kudos: 442
Location: India
Schools: ISB '21 (A)
GMAT 1: 570 Q43 V26
GMAT 2: 690 Q50 V32
GPA: 3.8
Send PM
Re: Veritas Prep CR Forum Expert - Karishma - Ask Me Anything about CR [#permalink]
Gladiator59 wrote:
Dear VeritasKarishma,

Whenever I run into CR trouble I turn to you. :-)

Can you please comment on this one - https://gmatclub.com/forum/in-a-recent- ... l#p2194617 . I am in a dilemma between option C vs. D

As always, you're the best. Thank you.

Regards,
Gladi


Hello VeritasKarishma Ma'am,

I have posted a small query to the same question on above forum after reviewing your answer. Thanks for the detail. I still had a small inch of doubt between C and D and hence I turned to you.
If you get time, will it be possible for you to review the same and let me know if the line-of-thinking what I have followed is proper?

Link: https://gmatclub.com/forum/in-a-recent- ... l#p2195011

The answer was correct, but I need to see if I got it correct for the right reasons.




Thanks & Regards,
Rishav
Tutor
Joined: 16 Oct 2010
Posts: 14817
Own Kudos [?]: 64906 [0]
Given Kudos: 426
Location: Pune, India
Send PM
Re: Veritas Prep CR Forum Expert - Karishma - Ask Me Anything about CR [#permalink]
Expert Reply
Gladiator59 wrote:
Dear VeritasKarishma,

Whenever I run into CR trouble I turn to you. :-)

Can you please comment on this one - https://gmatclub.com/forum/in-a-recent- ... l#p2194617 . I am in a dilemma between option C vs. D

As always, you're the best. Thank you.

Regards,
Gladi


Gladi, you are most welcome to send me your queries. Here is the answer to this one:
https://gmatclub.com/forum/in-a-recent- ... l#p2194821
Director
Director
Joined: 03 Mar 2017
Posts: 586
Own Kudos [?]: 418 [0]
Given Kudos: 596
Location: India
Concentration: Operations, Technology
Send PM
Re: Veritas Prep CR Forum Expert - Karishma - Ask Me Anything about CR [#permalink]
Hi VeritasKarishma,

Thanks a lot for addressing all the previous queries of mine.

Please help me to understand why option A is correct.

https://gmatclub.com/forum/last-august- ... 60887.html

Regards,
Prerak
GMAT Club Bot
Re: Veritas Prep CR Forum Expert - Karishma - Ask Me Anything about CR [#permalink]
   1  ...  5   6   7   8   9   10   11  ...  34   
Moderators:
GMAT Club Verbal Expert
6920 posts
GMAT Club Verbal Expert
238 posts
CR Forum Moderator
832 posts

Powered by phpBB © phpBB Group | Emoji artwork provided by EmojiOne